mrudula_2005
Thanks Received: 21
Forum Guests
 
Posts: 136
Joined: July 29th, 2010
 
 
trophy
First Responder
 

Q19 - Columnist: Much of North America

by mrudula_2005 Thu Aug 19, 2010 5:57 pm

Hi,

How does D, if true, not strengthen the columnist's reasoning?

Thanks
 
giladedelman
Thanks Received: 833
LSAT Geek
 
Posts: 619
Joined: April 04th, 2010
 
 
 

Re: Q19 - Columnist: Much of North America

by giladedelman Fri Aug 20, 2010 6:23 pm

Can you explain why you think (D) does strengthen the reasoning? On the face of it, I don't see how it affects the claim that excessively restrictive policies would diminish the wealth necessary to adopt the good policies. For one thing, we have no idea what the "compromise" mentioned would actually constitute. Would it be less restrictive or more restrictive? We have no idea.
 
mrudula_2005
Thanks Received: 21
Forum Guests
 
Posts: 136
Joined: July 29th, 2010
 
 
trophy
First Responder
 

Re: PT45, S1, Q19 - Much of North America and western Europe

by mrudula_2005 Fri Aug 20, 2010 6:29 pm

giladedelman Wrote:Can you explain why you think (D) does strengthen the reasoning? On the face of it, I don't see how it affects the claim that excessively restrictive policies would diminish the wealth necessary to adopt the good policies. For one thing, we have no idea what the "compromise" mentioned would actually constitute. Would it be less restrictive or more restrictive? We have no idea.


I just realized that I added in my own assumption that the compromise would be more restrictive - wrong move, for sure. I actually see how it does nothing to strengthen the argument now. Thanks!
 
Gabyn3
Thanks Received: 0
Vinny Gambini
Vinny Gambini
 
Posts: 4
Joined: October 01st, 2010
 
 
 

Re: PT45, S1, Q19 - Much of North America and western Europe

by Gabyn3 Sat Jan 15, 2011 10:07 pm

Can you explain why A and not E? Thanks.
User avatar
 
ManhattanPrepLSAT1
Thanks Received: 1909
Atticus Finch
Atticus Finch
 
Posts: 2851
Joined: October 07th, 2009
 
 
 

Re: PT45, S1, Q19 - Much of North America and western Europe

by ManhattanPrepLSAT1 Thu Jan 20, 2011 5:03 am

I was just explaining this one to a group of students in class the other night. Thanks for keeping the discussion going!

The simple answer is that answer choice (A) is correct because it strengthens the argument, and answer choice (E) is not correct because it weakens the argument.

The conclusion is that the strength of the environment lends credibility to the argument that environmental policies that excessively restrict the use of natural resources may diminish the wealth necessary to adopt and and sustain such policies. It's a mouthful, but it says that environmental policies will reduce our wealth.

Answer choice (A) shows how environmental policies will impact our wealth. It says that our wealth is mostly generated from exploiting the environment. How could we then be wealthy if we enacted all sorts of environmental policies restricting our ability to exploit the environment?

Answer choice (E) weakens the argument by showing that even if we enacted environmental policies, we could still be wealthy.

Does that answer your question?
 
clarafok
Thanks Received: 5
Forum Guests
 
Posts: 98
Joined: December 27th, 2010
 
 
trophy
Most Thankful
trophy
First Responder
 

Re: PT45, S1, Q19 - Much of North America and western Europe

by clarafok Thu Jan 20, 2011 11:41 am

wow i had a lot of trouble making sense of the last sentence but you make it seem so simple!

so in a gist, the question had nothing to do with the first sentence? and the part about policies advocated by environmentalists??

thanks!
User avatar
 
LSAT-Chang
Thanks Received: 38
Atticus Finch
Atticus Finch
 
Posts: 479
Joined: June 03rd, 2011
 
 
trophy
Most Thankful
trophy
First Responder
 

Re: PT45, S1, Q19 - Much of North America and western Europe

by LSAT-Chang Mon Jul 25, 2011 8:57 pm

mshermn Wrote:I was just explaining this one to a group of students in class the other night. Thanks for keeping the discussion going!

The simple answer is that answer choice (A) is correct because it strengthens the argument, and answer choice (E) is not correct because it weakens the argument.

The conclusion is that the strength of the environment lends credibility to the argument that environmental policies that excessively restrict the use of natural resources may diminish the wealth necessary to adopt and and sustain such policies. It's a mouthful, but it says that environmental policies will reduce our wealth.

Answer choice (A) shows how environmental policies will impact our wealth. It says that our wealth is mostly generated from exploiting the environment. How could we then be wealthy if we enacted all sorts of environmental policies restricting our ability to exploit the environment?

Answer choice (E) weakens the argument by showing that even if we enacted environmental policies, we could still be wealthy.

Does that answer your question?


Hi Matt!

Why can't (D) strengthen the argument??
The author is basically saying that these environmental improvenments should also be credited to the people who went against the environmental policies (thus, there not being a "compromise" between the two) -- so I would assume that if the two HAD compromised and therefore if there were no opposing sides, then the environment would have seen significantly less improvement. That would strengthen the argument.. would it not?

I may be misunderstanding something here.. but that was what I thought even after reading all the explanations above..

OR.. since the author's conclusion is just that "it lends credibility to the claims of the people who are against the environmental policies".. does it not matter about compromising or not?? Since the author isn't arguing that because the two did NOT compromise with one another, the environment is better -- but rather arguing that we should give credit to the other folks -- and thus, (A) supports the idea that in fact, these "other folks" claims are RIGHT -- therefore strengthens the idea that we SHOULD give credit to the other folks (felt like i just said that over and over again).

I love how I can get my thought together by spitting out my thought in words in this forum -- i didn't realize that it would help so much explaining my thought process to someone else. There have been at least 5 other posts that I wrote about questions in which I came to realize by myself after describing my thought process in words that I was wrong (just like this one -- hopefully). Does my explanation qualify? If you want to add anything else about why (D) does not strengthen just to clarify my understanding, please do so!! I would appreciate any other thoughts!
User avatar
 
ManhattanPrepLSAT1
Thanks Received: 1909
Atticus Finch
Atticus Finch
 
Posts: 2851
Joined: October 07th, 2009
 
This post thanked 2 times.
 
 

Re: Q19 - Much of North America and western Europe

by ManhattanPrepLSAT1 Tue Jul 26, 2011 3:42 am

So here's what's going on with answer choice (D). We have a special name for this sort of incorrect answer when we're dealing with Strengthen questions. We call this a premise booster. Answer choice (D) simply supports the claim that there has been an improvement in air quality and the levels of acid rain compared to what otherwise would have occurred.

But that's not the conclusion of the argument! The key is to focus on the conclusion. Supporting a premise of an argument does not actually support an argument upon which a premise is based. The logical validity of an argument is measured by determining whether the conclusion is guaranteed assuming the evidence is true. In essence, we already assumed the evidence is true, there's no need to now support it.

The conclusion of the argument is about whether the environmental policies will diminish the wealth needed to adopt and sustain such policies. Answer choice (A) addresses the issue of wealth head on and connects it to the use of resources, which was connected to the environmental policies in the argument itself.

Make sense?
User avatar
 
LSAT-Chang
Thanks Received: 38
Atticus Finch
Atticus Finch
 
Posts: 479
Joined: June 03rd, 2011
 
 
trophy
Most Thankful
trophy
First Responder
 

Re: Q19 - Much of North America and western Europe

by LSAT-Chang Wed Jul 27, 2011 4:18 pm

mshermn Wrote:So here's what's going on with answer choice (D). We have a special name for this sort of incorrect answer when we're dealing with Strengthen questions. We call this a premise booster. Answer choice (D) simply supports the claim that there has been an improvement in air quality and the levels of acid rain compared to what otherwise would have occurred.

But that's not the conclusion of the argument! The key is to focus on the conclusion. Supporting a premise of an argument does not actually support an argument upon which a premise is based. The logical validity of an argument is measured by determining whether the conclusion is guaranteed assuming the evidence is true. In essence, we already assumed the evidence is true, there's no need to now support it.

The conclusion of the argument is about whether the environmental policies will diminish the wealth needed to adopt and sustain such policies. Answer choice (A) addresses the issue of wealth head on and connects it to the use of resources, which was connected to the environmental policies in the argument itself.

Make sense?


Makes perfect sense! I don't know why I keep falling into answer choices that are labeled as "premise boosters".. also I think I was too focused into the author claiming "it lends credibility to the claims of people..." which made me think that that was the most important part of the argument, when in fact, it was all about the wealth (which is the actual "claim" made by those people and not the fact about how it lends credibility to the claim).
 
mcgarrk
Thanks Received: 0
Forum Guests
 
Posts: 5
Joined: May 21st, 2012
 
 
 

Re: Q19 - Columnist: Much of North America

by mcgarrk Tue May 29, 2012 7:44 pm

I'm confused. Various posts have mentioned their concerns about whether (D) strengthens the argument. Mshermn labels it a premise booster, "Answer choice (D) simply supports the claim that there has been an improvement in air quality and the levels of acid rain compared to what otherwise would have occurred."

But it seems clear to me that, if anything, (D) could weaken. Granted, you would have to make an assumption (never a good thing to do), but I wanted to make sure my logic/understanding of this answer isn't fundamentally flawed.

I assume that the "proponents of economic growth" from the ans and the "people who... argue that environmental policies excessively restrict" from the arg are more or less interchangeable groups. If that is the case, then it seems to me that (D) would weaken, since it would demonstrate that environmental policies DON'T in fact excessively restrict. Instead, when compromises are made regarding the policies, the environment saw "significantly less improvement". Granted, you have to assume that the compromise did in fact compromise the environmentalists policies in favor of the economic policies, but this seems to be common sense, given what the argument tells us about each group's views. Thus, the policies are NOT excessively restrictive, but in fact at least an important part of the environmental improvement.

Thoughts?
User avatar
 
ManhattanPrepLSAT1
Thanks Received: 1909
Atticus Finch
Atticus Finch
 
Posts: 2851
Joined: October 07th, 2009
 
 
 

Re: Q19 - Columnist: Much of North America

by ManhattanPrepLSAT1 Thu May 31, 2012 6:56 pm

I see it differently mcgarrk. I don't see the conclusion being that environmental policies excessively restrict the use of natural resources. Instead the conclusion is that environmental policies that excessively restrict the use of natural resources diminish the wealth necessary to adopt and sustain the policies that brought about the improvements.

The difference is that the subject changes to discussing those environmental policies that restrict the use of natural resources and not making a claim about environmental policies in general.

What do you think?
 
mcgarrk
Thanks Received: 0
Forum Guests
 
Posts: 5
Joined: May 21st, 2012
 
 
 

Re: Q19 - Columnist: Much of North America

by mcgarrk Sat Jun 02, 2012 3:38 pm

Even given the conclusion (I agree with you there), how is D a premise BOOSTER? Basically, I see the argument as:

Since the environment is doing better now than it was 5 decades ago,
So we can reject claims of ecological doom,
---------------------------------------------------
Thus, "these policies are excessively restrictive and diminish the wealth necessary to adopt and sustain the policies that brought about the improvements."

The justification for the claim that the policies diminish wealth directly hinges on this idea of being "excessively restrictive", since we can't justify shortening the budget on environmental policies if they are necessary for preventing imminent ecological doom (who cares about our national wealth if the world is coming to an end?). But since there is no chance of ecological doom right now, because the enviro has improved significantly, the policies are excessively restrictive. We could be spending less and making more, without chance of ecological doom, if only we made them less restrictive.

So what if we add D to the picture? D would be the scenario if, 5 decades ago, the economists got their way, and a "compromise" was made the enviro policies (we assume) less restrictive. Now the environment is doing significantly less better.

Wouldn't that at least weaken (as opposed to boost) the first premise (that the environment is doing better now than it was 5 decades ago) - oops. Wait a minute. I was reading ans D to mean that the environment had then declined or at least HADN'T improved at all, which would then, I think, weaken their assumption of excessive restriction, and thus weaken the main conclusion about the (unnecessary) consumption of wealth. Because, after all, if the environment isn't improving, or is declining, isn't it a little more difficult to rule out ecological doom? And thus a little harder to justify making policies less restrictive, etc.?

But, I just noticed that Ans D says, "would have seen significantly less IMPROVEMENT", meaning that even in that scenario, the economists could still argue that the environment had IMPROVED (albeit LESS in this scenario) and thus there still wasn't a chance of imminent ecological doom, thus still excessive restriction, and so on. Even if, in reality, improvement from ans D's scenario is only 1% better, instead of 50% better, a small improvement is still, they could argue, an improvement. So, D doesn't really weaken - but... I still would argue it isn't "boosting" either... it seems to be more neutral to me. Not sure.

Did any of that make sense? Thoughts?
 
acechaowang
Thanks Received: 4
Vinny Gambini
Vinny Gambini
 
Posts: 23
Joined: July 03rd, 2012
 
 
 

Re: Q19 - Columnist: Much of North America

by acechaowang Fri Sep 21, 2012 10:33 pm

hi, this problem troubles me a lot as well. But after delving into it, i think i got why D is wrong. THe way how I look at it is this: the conclusion is that the better environment is due to the claims of these people. So I think of this argument as a causal relation. A strenthens the argument by providing additional evidence to assert that these people's belief is indeed true, that is, the cause is indeed correct. D is worse choice compared to A since D does not touch on the conclusion. Even though there might be a compromise, there claims could still exist and thus it could actually weaken the argument since claims exist, but the environment becomes less improved. Therefore, A is the right answer. hope it helps!
 
stm_512
Thanks Received: 0
Vinny Gambini
Vinny Gambini
 
Posts: 17
Joined: June 24th, 2014
 
 
 

Re: Q19 - Columnist: Much of North America

by stm_512 Mon Aug 11, 2014 7:56 pm

I think A) is correct for a reason in addition to what Matt mentioned.

The argument is basically: excessively restricting the use of natural resources leads to less money available to pursue policies that can improve the environment. So it's saying we actually need to exploit some natural resources in order to bring about environmental improvements via the wealth creation associated with the resource exploitation.

A) Not only suggests that nations sustain wealth through using natural resources, it also states that they are found within their boundaries.

If A) is not true, i.e. if nations can sustain their wealth via sources other than using natural resources within their boundaries (for example trade without national resource commodities), then it would weaken the argument -you can greatly restrict the use of natural resources and still gain wealth necessary to bring long-term environmental improvements.

In this light, A) is strengthening the argument by defending its possible weakness.
 
gaheexlee
Thanks Received: 10
Elle Woods
Elle Woods
 
Posts: 55
Joined: May 27th, 2014
 
 
 

Re: Q19 - Columnist: Much of North America

by gaheexlee Wed Nov 19, 2014 1:30 pm

One way to eliminate (D) is to recognize its vagueness:

(D) says that a compromise would have led to less environmental improvement. But it doesn't tell us why.

For (D) to be the correct answer, we as readers would have to assume that the compromise would have been less effective because it wouldn't have allowed the economy to boom as much, thus leading to less resources to devote to environmental protectionism. But who's to say that the compromise's disappointing impact wouldn't have stemmed from a disagreement between the two groups about why dogs make better pets than cats?

In short, (D) is too vague to be a true strengthener. We should never have to supply our own (unwarranted) assumptions to justify an answer's correctness!
 
asafezrati
Thanks Received: 6
Atticus Finch
Atticus Finch
 
Posts: 116
Joined: December 07th, 2014
 
 
 

Re: Q19 - Columnist: Much of North America

by asafezrati Fri Jul 31, 2015 9:22 pm

I agree with the above post that D is vague. It implies that if decisions in the past had been less restrictive the environment could have been expected to be in a worse state. But the environment currently being in a good state somehow lends credibility to the anti-environmental people who focus on wealth.

The problem for me was to understand the core.
It seems that the core is as follows:

Premise:
Environment has improved in the past 50 years
Conclusion:
(even though it happened due to environmental policies) this improvement indicates that:
[environmental restrictions -> less wealth -> harder to maintain environmental policies]

I reached A only because the others seemed wrong, but I don't see how it relates to the reasoning. It seems that it's directed at the conclusion itself, and doesn't deal with the connection between the premise (good environment) and the conclusion (environment->less wealth->less environment).

Any thoughts on this?
User avatar
 
rinagoldfield
Thanks Received: 309
Atticus Finch
Atticus Finch
 
Posts: 390
Joined: December 13th, 2011
 
 
 

Re: Q19 - Columnist: Much of North America

by rinagoldfield Sat Aug 08, 2015 5:08 pm

Agreed with gaheexlee and gilad that (D) is too vague. Does the compromise mean more environmentally restrictive or less? We don’t know. If the outcome is a worse environment, is that due to a lack of economic growth or a lack of environmental regulations? Again, we don’t know. There’s way too much missing from (D) for it to strengthen.

asafezrati,

Here’s how I’d frame the core:

Environment improve
-->
Reject predictions of economic doom! We can diminish environmental regulations and work on wealth-building.
 
IrisH894
Thanks Received: 0
Vinny Gambini
Vinny Gambini
 
Posts: 11
Joined: September 13th, 2022
 
 
 

Re: Q19 - Columnist: Much of North America

by IrisH894 Mon Sep 19, 2022 9:23 am

D is not a premise booster.
According to the stimuli, what happened 50 years ago was that environmentalists got their way and the environment got better because of the policies they advocated (i.e. the current policies).
What the conclusion of the argument argues against, is a scenario where more restrictive policies were enacted. This is bad because we would have lost the money necessary to enact the effective policies that are currently in place.
So we have no idea where proponents of economic growth stand in terms of environmental policies. They are just saying that more restrictive than status quo = bad. Because of the uncertainty this answer choice is immediately eliminated.